Find F-1(x), the inverse of F(x), for 3 and 4

Find F-1(x), The Inverse Of F(x), For 3 And 4

Answers

Answer 1

Solving for number 3:

Step 1. Define the function.

The original function we have is:

[tex]F(x)=x-10[/tex]

Step 2. To find the inverse function, the first thing we need to do is to change F(x) for y:

[tex]y=x-10[/tex]

Step 3. Now, we are going to interchange the letters x and y (swap x and y):

[tex]x=y-10[/tex]

Step 4. Finally, the last step is to solve for y:

[tex]y=x+10[/tex]

change y for F-1(x), and the inverse function is:

[tex]F^{-1}(x)=x+10^{}[/tex]

Answer:

[tex]F^{-1}(x)=x+10[/tex]

Answer 2

Answer:

Solving for number 3:

Step 1. Define the function.

The original function we have is:

Step 2. To find the inverse function, the first thing we need to do is to change F(x) for y:

Step 3. Now, we are going to interchange the letters x and y (swap x and y):

Step 4. Finally, the last step is to solve for y:

Change y for F-1(x), and the inverse function is:

Answer:

Step-by-step explanation:


Related Questions

If y varies inversely as x and y = 41 when x = 28, find y if x = 27. (Round off your answer to the nearest hundredth.)Answer How to enter your answer (Opens in new window) 6 Pointsy = 0y

Answers

When y varies inversely as x:

[tex]y=\frac{k}{x}[/tex]

y= 41 when x=28; uses the given data to find k:

[tex]\begin{gathered} 41=\frac{k}{28} \\ \\ k=41*28 \\ \\ k=1148 \end{gathered}[/tex]

Use the next formula to the given variation:

[tex]y=\frac{1148}{x}[/tex]

Find y if x=27:

[tex]\begin{gathered} y=\frac{1148}{27} \\ \\ y=42.52 \end{gathered}[/tex]Answer: y=42.52

At what value of w does the graph have a vertical asymptote? Explain how you know and what this asymptote means in the situation.

Answers

Vertical asymptote are vertical lines which corresponds to the zeros of the denominator of our rational function. Then, the zeros of T(w) ocurr when

[tex]530-w=0[/tex]

which gives

[tex]w=530[/tex]

Therefore, the vertical asymptote is w = 530.

The asymptote is a line that the graph function approaches but never touches. In our case, this means that the speed of the wind is very close to 530 mph but never touches this value.

while free diving in the ocean, Tanya streeter once said a record by diving 525 ft and 3 and 1/2 minutes. how many feet per minute did she dive?

Answers

ok

length = 525 ft

time = 3.5 min Is that correct 3 and 1/2 minutes?

rate = 525/3.5

rate = 150 ft/min

If a card is drawn from an ordinary deck of 52 cards, find the probability of getting a heart or a face card?

Answers

Given:

Total number of cards = 52

Number of face cards = 12

Number of heart cards = 13

Required: Probability of getting a heart or a face card

Explanation:

Number of cards that is both face cards and heart cards = 3

Number of cards that are either heart or face card

= 12 + 13 - 3

=22

Probability of getting a heart or face card = Number of favorable cases/ Total number of cases

[tex]=\frac{22}{52}=\frac{11}{26}[/tex]

Final Answer: The probability of getting a heart or face card is 11/26.

I need help with solving propotional segments in right triangles. I need help with number 4.

Answers

We have

AD=9 in

DB=4 in

find DC,AC and BC

we can do the next equivalences

[tex]\frac{AD}{DC}=\frac{DC}{DB}[/tex][tex]\frac{9}{DC}=\frac{DC}{4}[/tex]

then we clear DC

[tex]\begin{gathered} DC=\sqrt[]{36} \\ DC=6 \end{gathered}[/tex]

For AC we use the Pythagorean theorem

[tex]AC=\sqrt[]{9^2+6^2}=10.81[/tex]

For BC we use the Pythagorean theorem

[tex]BC=\sqrt[]{6^2+4^2}=7.21[/tex]

Please help me if you could if you can't I understand. what fractions are equivalent to 2/3 and 7/12 using the least common denominator?

Answers

2/3 ---->8/12

7/12 ----> 7/12

1) Equivalent fractions have the same value proportionally, so let's find out equivalent fractions:

[tex]\frac{2}{3}+\frac{7}{12}[/tex]

2) To find equivalent fractions and sum those fractions, let's factorize 3 and 12 dividing them only by Prime Numbers, when one of those numbers can't be divided then we repeat it below:

As we can see on the first line, 12 can be divided by 2 and 3 cannot.

So we repeat 3 on the line below.

We then picked 6 and divided by 2, and then repeated below 3.

Then divided3 by 3

3) Now we can rewrite 2/3 + 7/12 as:

So using the Least Common Denominator we have 2/3 (8/12) and 7/12 (7/12) as their equivalent fractions. Note that 7/12 in this case is equivalent to itself.

An 8-pack of batteries cost $9.44.what is the price, in dollars , of one battery?A)- $0.85B)- $1.18C)- 1.44D)- 2.36

Answers

In order to determine the cost of only one battery, calculate the quotient in between the cost of 8 batteries over 8:

$9.44/8 = $1.18

Hence, one battery costs $1.18

question number 1 and 2 and find measure of. angle 1

Answers

[tex]\begin{gathered} 1)\text{ vertical} \\ m\measuredangle HML\text{ and m}\measuredangle JMK\text{ , } \\ m\measuredangle LQM\text{ and m}\measuredangle\text{PQN} \\ 2)\text{ adjacents} \\ m\measuredangle HMJ\text{ and m}\measuredangle JMK \\ m\measuredangle LQM\text{ and m}\measuredangle LQR \end{gathered}[/tex]

Explanation

Step 1

vertical angles:

Vertical angles are pair angles formed when two lines intersect

[tex]m\measuredangle x=m\measuredangle y[/tex]

so, we need to find a vertical angle in

a)

Figure 1:

blue angles are vertical, so

[tex]m\measuredangle HML\text{ and m}\measuredangle JMK[/tex]

Figure 2:

hence, a pair of vertical angle is

[tex]\begin{gathered} \\ m\measuredangle LQM\text{ and m}\measuredangle\text{PQN} \end{gathered}[/tex]

Step 2

pair of adjacent angles:

Adjacent angles are two angles that have a common vertex and a common side but do not overlap

[tex]m\measuredangle x\text{ is adjacent to m}\measuredangle y[/tex]

then

a)

for Figure 1

pair of adjacent angles

[tex]m\measuredangle HMJ\text{ and m}\measuredangle JMK[/tex]

b) for Figure 2

pair of adjacent angles

[tex]m\measuredangle LQM\text{ and m}\measuredangle LQR[/tex]

I hope this helps you

Write the nth rule for each geometric sequence.5) 7, 14, 28, 56...

Answers

We have the next numbers

[tex]7,14,28,56[/tex]

as we can see we have double of the previous number, so the rule is

[tex]a_n=a_{n-1}\cdot2[/tex]

we need to prove the rule

[tex]a1=7[/tex]

[tex]a2=7\cdot2=14[/tex][tex]a3=14\cdot2=28[/tex][tex]a4=28\cdot2=56[/tex]

as we can see the rule is appropriate for the geometric sequence

PLEASE HELP!! ALGEBRA 1 HW I WILL GIVE BRAINLYEST​

Answers

Answer:

answer Is -5/2 becouse if the line Is horizontal his equation Is y=k and in this case k= -5/2

El producto de 2 por la diferencia de x y y

Answers

La diferencia de 'x' y 'y' se puede escribir asi:

[tex]undefined[/tex]

Hello. I think that I'm overthinking this. I'm pretty sure it's a monomial?

Answers

The expression 5x⁶ - x⁴ is a binomial because we have two terms.

Even if they have the same variable x, their exponents are not the same.

For the school play, tickets cost $13.50 for adults and $5 for kids under 12. How
many total tickets would someone get if they purchased 6 adult tickets and 22 kids
tickets? How many total tickets would someone get if they purchased a adult tickets
and k kids tickets?
Total tickets, 6 adult tickets and 22 kids tickets:
Total tickets, a adult tickets and k kids tickets:

Answers

Answer

The adult-135 And the Kid-27:

Step-by-step explanation:

13.50 * 10=135

4.50 * 6 =27

I need help with this 6-9 should be matched with either A-H

Answers

Explanation

To answer the question, we will make use of some of the properties of a parallelogram

These are

The opposite sides are parallel.

Opposite sides are congruent.

Opposite angles are congruent.

Same-Side interior angles (consecutive angles) are supplementary.

Each diagonal of a parallelogram separates it into two congruent triangles.

The diagonals of a parallelogram bisect each other.

Therefore

For question 6

[tex]\begin{gathered} mTherefore, the answer to question 6 is A

Question 7

[tex]mThe answer to question 7 is E

Question 8

[tex]\begin{gathered} DF=FB \\ Diagonals\text{ bisect each other} \\ DF=17 \end{gathered}[/tex]

The answer to question 8 is C

Question 9

[tex]\begin{gathered} mThe answer to question 9 is F

donuts at Krispy Kreme are always perfectly round. The diameter of the circular donut is 6 inches. Which of the following is closest to the circumference of the donut?

Answers

The circumference of a donut is computed as follows:

[tex]C=\pi\cdot D[/tex]

where D is the diameter of the donut. Substituting with D = 6,

[tex]\begin{gathered} C=\pi\cdot6 \\ C=18.85\text{ in} \end{gathered}[/tex]

Find the equation of the line, in slope-intercept form, through (-4, 6)and parallel to y=-3x + 4. (Show work) (3 pts)

Answers

The given equation is

[tex]y=-3x+4[/tex]

Notice that this equation represents a line in slope-intercept form, where its slope is -3.

Now, we have to find a new line parallel to the one above, which means the slope of the new line is also 3 because parallel lines have equal slopes.

Then, we use the given points (-4,6) and the slope -3 to find the equation

[tex]\begin{gathered} y-y_1=m(x-x_1) \\ y-6=-3(x-(-4)) \\ y-6=-3x-12 \\ y=-3x-12+6 \\ y=-3x-6 \end{gathered}[/tex]Therefore, the equation of the parallel line is y = -3x - 6.

Hi,I need help with this I tried and I can’t get the answer

Answers

Given

[tex]12000\div300[/tex]

We can decompose the above expression below

Explanation

300 can be decomposed into

[tex]300=100\times3[/tex]

Therefore,

Answer:

[tex]12000\div100\div3[/tex]

Saira is using the formula for the area of a circle to determine the value of .

Answers

For this problem, we just have to use the values we're given to calculate the approximate value of pi.

The formula presented is

[tex]\pi=Ar^{-2}[/tex]

When you have a negative exponent, we can use the following property

[tex]a^{-b}=\frac{1}{a^b}[/tex]

Using this property, our problem turns out to be

[tex]\pi=\frac{A}{r^2}[/tex]

Now, we just need to plug the given values on this equation

[tex]\pi=\frac{50.265}{4^2}=3.1415625\approx3.142[/tex]

The approximated value for pi is 3.142.

Alan bought 36 gallons of gas for $2.05 per gallon. How much did he spend ongas? *

Answers

Alan bought 36 gallons of gas for $2.05 per gallon.

total amount of gas = 36 gallons

Price of one gallon = $2.05

So, for the price of 36 gallons : Multiply the amount of 1 gallon with the the number of gallon

Price of 36 gallons = 36 x 2.05

Price of 36 gallons = $73.8

Alan spend an amount of $73.8 on the gas of 36 gallons

Answer : $73.8

Write the ratio as a fraction in simplest form, with whole numbers in the numerator and denominator.0.50 : 0.25

Answers

Answer:

2/1

Explanation:

Given the ratio:

[tex]0.50\colon0.25​[/tex]

Divide both sides by 0.25

[tex]\begin{gathered} \frac{0.50}{0.25}\colon\frac{0.25​}{0.25} \\ =2\colon1 \\ =\frac{2}{1} \end{gathered}[/tex]

Thus, the ratio as a fraction in simplest form is 2/1.

PLEASE just give me the answers and not a whole defintion of every single word. I just want quick answers so I can check my work. *don't worry, this is just a math practice

Answers

7. m and n are parallel because both alternate interior angles are equal.

8.m and n are parallel because Alternate exterior angles are equal.

9.m and n are parallel Because corresponding angles are equal.

10. m and n are parallel because corresponding and consecutive angles are equal.

11. m and n are parallel because alternate exterior angles are equal.

12.m and n are parallel because vertical (opposite) angles are equal.

For what values of k will the sum of the solutions of x^2 - (k^2 - 3k)x + 24=0 be 10?

Answers

The Solution:

Given the equation below:

[tex]x^2-(k^2-3k)x+24=0[/tex]

We are required to find the value of k that will make the sum of the solutions to be 10.

Step 1:

Let:

[tex]\begin{gathered} k^2-3k\text{ be represented with b} \\ \text{ So that we have} \\ k^2-3k=b\ldots eqn(1) \end{gathered}[/tex]

So, the given equation becomes:

[tex]x^2-bx+24=0[/tex]

We shall the Quadratic Formula Method to solve for x in terms of b.

In this case,

[tex]\begin{gathered} a=1 \\ b=-b \\ c=24 \end{gathered}[/tex]

Substituting, we get

[tex]x=\frac{-b\pm\text{ }\sqrt[]{(-b)^2-(4\times1\times24)}}{2(1)}[/tex][tex]x=\frac{-b\pm\text{ }\sqrt[]{b^2-96}}{2}[/tex]

So, the solutions to the given equation are:

[tex]\begin{gathered} x=\frac{-b+\text{ }\sqrt[]{b^2-96}}{2} \\ \text{ or} \\ x=\frac{-b-\text{ }\sqrt[]{b^2-96}}{2} \end{gathered}[/tex]

Equating their sum to 10.

[tex]\begin{gathered} \frac{-b+\text{ }\sqrt[]{b^2-96}}{2}+\frac{-b-\text{ }\sqrt[]{b^2-96}}{2}=10 \\ \\ \\ \frac{-b+\text{ }\sqrt[]{b^2-96}+-b-\text{ }\sqrt[]{b^2-96}}{2}=10 \end{gathered}[/tex]

Simplifying, we get

[tex]\begin{gathered} \frac{-2b}{2}=10 \\ \\ -b=10 \end{gathered}[/tex]

Substituting for b, we get

[tex]\begin{gathered} -(k^2-3k)=10 \\ k^2-3k=-10 \\ k^2-3k+10=0 \end{gathered}[/tex]

Solving for k by the Quadratic Formula method of solving quadratic equation, we get

[tex]k=\frac{-b\pm\text{ }\sqrt[]{b^2-4ac}}{2a}[/tex]

Where

[tex]a=1,b=-3\text{ and c=10}[/tex]

Substituting, we get

[tex]k=\frac{-(-3)\pm\text{ }\sqrt[]{(-3)^2-(4\times1\times10)}}{2(1)}[/tex][tex]k=\frac{3\pm\text{ }\sqrt[]{9^{}-40}}{2}=\frac{3\pm\text{ }\sqrt[]{-31}}{2}[/tex][tex]\begin{gathered} k=\frac{3+\text{ }\sqrt[]{-31}}{2}\text{ or }k=\frac{3-\text{ }\sqrt[]{-31}}{2} \\ \end{gathered}[/tex]

Therefore, the correct answer is

[tex]k=\frac{3+\text{ }\sqrt[]{-31}}{2}\text{ or }k=\frac{3-\text{ }\sqrt[]{-31}}{2}[/tex]

Alternatively,

We can use the sum of roots formula below:

[tex]\begin{gathered} \text{ Sum of roots = }\frac{-b}{a} \\ \text{if given a quadratic equation of the form ax}^2+bx+c=0 \end{gathered}[/tex]

So, we get

[tex]\begin{gathered} a=1 \\ b=-(k^2-3k) \\ c=24 \end{gathered}[/tex]

So,

[tex]\begin{gathered} \text{ Sum=}\frac{--(k^2-3k)}{1}=10 \\ \\ k^2-3k=10 \\ \\ k^2-3k-10=0 \end{gathered}[/tex]

Then you can now solve from here as have done in the previous method.

Solve the quadratic equation above for k.

20. A sequence is defined recursivelybelow:+4a, = 8,-1a, = -3Which function can be used to find the nthsequence?

Answers

A)

1) Let's insert the Recursive formula for a:

[tex]\begin{gathered} a_n=a_{n-1}+4 \\ a_1=-3 \\ f(n)=-3\text{ +(n-1)4} \\ f(n)=-3\text{ +4n-4} \\ f(n)=4n-7 \end{gathered}[/tex]

When we apply the General formula f(n) we'll need the first term. for that sequence, and then rearrange it we find the function for the nth term of that Arithmetic Sequence.

All we need now is to plug into the formula the nth term we want to find out.

which methods correctly solve for variable x in the equation 3/4 (x - 8)=6?

Answers

SOLUTION

The equation given is

[tex]\frac{3}{4}(x-8)=6[/tex]

Step1

Multiply both sides by

[tex]\frac{4}{3}[/tex]

We have

[tex]\begin{gathered} \frac{4}{3}\times\frac{3}{4}(x-8)=\frac{4}{3}\times6 \\ \\ x-8=8 \end{gathered}[/tex]

Step2

Add 8 to both sides of the equation

[tex]\begin{gathered} x-8+8=8+8_{} \\ x=16 \end{gathered}[/tex]

Therefore, the correct method that solves the equation is

5. Multiply both sides by 4/3 and then add 8 to both sides of the equation

similarly,

Distribute 3/4 to (x-8)

[tex]\begin{gathered} \frac{3}{4}(x-8)=6 \\ \frac{3}{4}x-6=6 \end{gathered}[/tex]

Then add 6 to both sides

[tex]\begin{gathered} \frac{3}{4}x-6=6 \\ \\ \frac{3}{4}x-6+6=6+6 \\ \frac{3}{4}x=12 \end{gathered}[/tex]

Then multiply both sides by 4/3

[tex]\begin{gathered} \frac{3}{4}x\times\frac{4}{3}=12\times\frac{4}{3} \\ \\ x=16 \end{gathered}[/tex]

Hence

2. Distribute 3/4 to (x-8), then add 6 to both sides and finally multiply both sides by 4/3

Therefore

Distribute 3/4 to (x-8), then add 6 to both sides and finally multiply both sides by 4/3

and

You earn a salary of $40000 per year and decide to save 20% of your gross pay. You then set a goal of creating $16000 and emergency fund. How long will it take you to achieve your goal?

Answers

Gross pay amount = $40,000

Percentage of the gross pay saved = 20%

Amount of gross pay saved per year = 20% of 40,000

Amount of gross pay saved per year = 20/100 * 40,000

Amount of gross pay saved per year = 20*400 = $8000

This means that $8000 is saved per year.

Next is to calculate the number of years it will take to save $16,000

Let us use the equality postulate;

1 year = $8,000

x year = $16,000

cross multiply

8000x = 16000

x = 16000/8000

x = 2

This means that it will take 2 years for me to achive my goal.

Draw a graph of a parabola that has the followinsignificant features:a > 0one X - intercepty - intercept at 4Write the equation to your parabola:

Answers

[tex]y=x^2+4x+4[/tex]

What is the length of the side opposite the 30° angle? Explain your reasoning.

Answers

Given the triangle ABC as shown below:

The length of the side opposite the 30° angle is evaluated as follows:

Step 1:

Given that the 30° angle is the focus angle, label the sides of the triangle.

Thus,

[tex]\begin{gathered} \text{where }\theta=30^{\circ} \\ AC\Rightarrow hypotenuse\text{ (the longest side of the triangle)} \\ AB\Rightarrow opposite\text{ (the side opposite the focus angle)} \\ BC\Rightarrow adjacent \\ \text{thus, } \\ AC\text{ = 44} \\ AB\text{ = x (unknown length)} \end{gathered}[/tex]

Step 2:

Evaluate the unknown side using trignometric ratios.

By trigonometric ratios,

[tex]\begin{gathered} \sin \theta\text{ = }\frac{opposite}{hypotenuse}=\frac{AB}{AC} \\ \cos \text{ }\theta\text{ = }\frac{adjacent}{hyptenuse}=\frac{BC}{AC} \\ \tan \text{ }\theta\text{ = }\frac{opposite}{adjacent}=\frac{AB}{BC} \end{gathered}[/tex]

From the above trigonometric ratios, sine θ is used to evaluate the value of the unknown side.

This because the sine θ gives the ralationship between the hypotenuse and the unknown side of the triangle.

Thus,

[tex]\begin{gathered} \sin \theta\text{ = }\frac{opposite}{hypotenuse}=\frac{AB}{AC} \\ AB\text{ = x} \\ AC\text{ = 44} \\ \theta\text{ = 30} \\ \Rightarrow\Rightarrow\sin 30\text{ = }\frac{x}{44} \\ 0.5\text{ = }\frac{x}{44} \\ \Rightarrow x\text{ = 0.5}\times44 \\ x\text{ = 22} \end{gathered}[/tex]

Hence, the value of the unknown side is 22.

In circle O, mPN= 131°. Solve for x if m

Answers

SOLUTION

Given the question in the image, the following are the solution steps to answer the question.

STEP 1: state the required theorem

The arc measure is equal to the angle at the center. Arc measure is a degree measurement, equal to the central angle that forms the intercepted arc.

STEP 2: Get the equation

[tex]131^{\circ}=6x+26[/tex]

STEP 3: Solve for x

[tex]\begin{gathered} 131=6x+26 \\ 131-26=6x \\ 105=6x \\ \frac{6x}{6}=\frac{105}{6} \\ x=17.5 \end{gathered}[/tex]

Hence, the value of x is

Are these triangles congruent? If they are, justify it which congruence statement. If not, say cannot be determined. *

Answers

According to the figure given

We are given triangle GFH and EFH

[tex]\begin{gathered} <\text{ H }\cong\text{ < F} \\ FE\text{ }\cong\text{ GH} \end{gathered}[/tex]

Line FE is parallel to line GH

Therefore, the triangles are congruent by side and angle

Mike needs to calculate the angle a rafter makes a with a ceiling joist of a house. The roof has a rise of 5.5 for a run of 12’. What is the angle of the rafter ?

Answers

Let's illustrate the given information.

To determine the angle of this rafter, we can use the tangent function. The formula is:

[tex]\tan \theta=\frac{opposite\text{ side}}{\text{adjacent side}}[/tex]

Our angle in the illustration is the one colored in red. The opposite side of the angle measures 5.5 inches while the adjacent side measures 12 inches. Let's plug in this data to the formula above.

[tex]\tan \theta=\frac{5.5}{12}[/tex]

To be able to get the measure of angle, let's get the arc tan of the angle.

[tex]\theta=\tan ^{-1}\frac{5.5}{12}\approx24.62[/tex]

Hence, the rafter must be angled 24.62 degrees away from the ceiling joist of the house.

Other Questions
Perception is not based upon what does refractory period mean? 4. Felix and Oscar applied for the same credit card from the same bank.The bank checked both of their FICO scores. Felix had an excellentcredit rating, and Oscar had a poor credit rating.a. Felix was given a card with an APR of 12%. What was hismonthly percentage rate?b. Oscar was given a card with an APR of 15%. What was hismonthly payment?c. If each of them had an average daily balance of $800 and had topay a finance charge, how much more would Oscar pay than Felix? wilson wanted to create an organization of nations to work together to preserve world peace. group of answer choices true false Determine if each expression is equal to (2) (-3) (-4)(6) (-4)(-3) (8)(12) (2)(-4) (-6) What process can only be carried out byplants when in the presence of light?A. homeostasisB. light-dependent reactionsC. cellular respirationD. dark reactions how do I calculate the area of a partial circle? Write a summary at least 200 words long A theme is the main idea or message of a text.A theme must be a complete sentence.Bad Examples: Courage; Loyalty; Integrity; Nature Good Example: Forces of nature are stronger than man.Themes must apply outside of the story.Bad Example: Cory's loyalty caused him to make noble sacrifices.Good Example: Loyalty causes people to make noble sacrifices.Texts can have more than one theme.Example: A text where two friends encounter dangerous forces of nature and are loyal to each other could have a theme about loyalty as well as a theme about the strength of nature.Themes must be supported with evidence.Many different themes can be developed from a text as long as there is evidence in the text that supports each theme. the measure of two angle are (8x+24) and (12x-16) what is the value of x if theses angle ar congruent The table below shows the thickness of coins. Coin Thickness quarter i millimeters 12 millimeters dime nickel millimeters penny 13 millimeters Hailey stacks a dime on top of a penny. She estimates the thickness of the two coins to be less than 3 millimeters. Write a symbol (, or =) in the box to make the statement true. Then use the statement to tell whether Hailey's estimate is correct. 12 + 12 + 1 Is Hailey's estimate correct? I really sure what to do for this question some help would be greatly appreciated Which exponential function models this situation ? Can you please solve this equation and please explain to me ^step-by-step^ (this is my homework) 1. 2^2 * 3x^3y * 3x^3y=1. 2^2 * 3x^3*y * 3x^3*y= Solve using substitution. y = 7x + 3 y = 6x + 4(_ , _) Solve for g: g + 8 > 10 I need help checking to make sure my work is correct. Start with the basic function f(x) = 2x. If you have an initial value of 1, then you end up with the following iterations:f(1) = 2 x 1 = 2f^2 (1) = 2 x 2 x 1 = 4f^3 (1) = 2 x 2 x 2 x 1 = 8The question Part 1: If you continue the pattern, what do you expect would happen to the numbers as the number of iterations grows? Check your result by conducting at least 10 iterations. I put: f^4 (1) = 2 x 2 x 2 x 2 x 1 = 16f^5 (1) = 2 x 2 x 2 x 2 x 2 x 1 = 32f^6 (1) = 2 x 2 x 2 x 2 x 2 x 2 x 1 = 64f^7 (1) = 2 x 2 x 2 x 2 x 2 x 2 x 2 x 1 = 128f^8 (1) = 2 x 2 x 2 x 2 x 2 x 2 x 2 x 2 x 1 = 256f^9 (1) = 2 x 2 x 2 x 2 x 2 x 2 x 2 x 2 x 2 x 1 = 512f^10 (1) = 2 x 2 x 2 x 2 x 2 x 2 x 2 x 2 x 2 x 2 x 1 = 1024Part 2: Repeat the process with an initial value of -1. What happens as the number of iterations grows? it takes james 15 minutes to cut a log into 4 pieces. find how long it will take him to cut a log into 15 pieces. estimate the product by rounding to the nearest 10: 285676 *Will mark brainiest* Rectangle ABCD is rotated 90 clockwise about the origin to produce Rectangle A'B'CD' What is the length, in units of line segment CD'?